Skip to main content

Visit the website manishverma.site for latest posts, courses, admission & more.

${\sin ^2}{1^\circ } + {\sin ^2}{2^\circ } + ............ + {\sin ^2}{180^\circ } = ?$

The given expression can be written as,

$\frac{1}{2}\left[ {(1 - \cos {2^\circ }) + (1 - \cos {4^\circ }) + ................ + (1 - \cos {{360}^\circ })} \right]$

$ = \frac{1}{2}\left[ {(1 + 1 + ........ + 1) - (\cos {2^\circ } + \cos {4^\circ } + ........... + \cos {{360}^\circ })} \right]$

There are 180 terms in the series.

$ = \frac{1}{2}\left[ {180 - \frac{{\cos \left( {{2^\circ } + \frac{{180 - 1}}{2} \times {2^\circ }} \right)\sin \left( {\frac{{180 \times {2^\circ }}}{2}} \right)}}{{\sin \left( {\frac{{{2^\circ }}}{2}} \right)}}} \right]$

$ = \frac{1}{2}\left( {180 - \frac{{\cos {{181}^\circ }\sin {{180}^\circ }}}{{\sin {1^\circ }}}} \right) = 90$

Popular posts from this blog

A man starts walking from the point P (-3, 4) ....

A man starts walking from the point P (-3, 4), touches the x-axis at R, and then turns to reach at the point Q (0, 2). The man is walking at a constant speed. If the man reaches the point Q in the minimum time, then $50 [(PR)^2 + (RQ)^2 ]$ is equal to _ _ _ _ . Solution For time to be minimum at constant speed, the directions must be symmetric. In other words, the angles made by PR and RQ with the vertical must be the same just like in the law of reflection in optics. $tan \theta = \frac {MP}{MR} = \frac {NQ}{NR} $ $\Rightarrow \frac {3-r}{4} = \frac {r}{2}$ $\Rightarrow r=1 $ So, $R \equiv ( - 1,0)$ Now, $50(PR^2+RQ^2)=50[(4+16)+(1+4)]=1250$